ProfEd | Professional Education Reviewer LET

 https://www.calubian.com/p/professional-education-reviewer-let.html  


PROFESSIONAL EDUCATION 


1)  What type of test must a teacher use if she wants to test the students' ability to organize ideas?

a.  Multiple-choice type

b.  Short answer

c.  Essay

d.  Technical problem type


Study this group of tests which was administered with the following results, then answer the question.


Subject  |  Mean  |  SD  |  Joana's Score

Math  |  80  |  7.5  |  95

Science  |  49  |  3  |  60

Filipino  |  95  |  12  |  90


2)  In which subject/s did Joana performs best in relation to the performance of the group?

a.  Math

b.  Science

c.  Filipino

d.  Math & Filipino


3)  Which subject did Joana performs poorly in relation to the performance of the group?

a.  Math

b.  Science

c.  Filipino

d.  Math & Filipino


4)  Which group were scores less scattered?

a.  Math

b.  Science

c.  Filipino

d.  Math & Filipino


5)  Which group were scores most widespread?

a.  Math

b.  Science

c.  Filipino

d.  Math & Filipino


6)  Leah obtained a score of 95 in Chemistry multiple-choice test. What does this imply?

a.  He has a rating of 95.

b.  He answered 95 items in the test correctly.

c.  He answered 95% of the test item correctly.

d.  His performance is 5% better than the group.


7)  In his second item analysis, Teacher Dave found out that more from the lower group got the test item #10 correctly. This means that the test item ________.

a.  has a negative discriminating power

b.  has a lower validity

c.  has a positive discriminating power

d.  has a high reliability


8)  NSAT and NEAT results are interpreted against a set mastery level. This means that NSAT and NEAT fall under ________.

a.  intelligence test

b.  aptitude test

c.  criterion-referenced test

d.  norm-referenced test


9)  This is the first step in planning an achievement test?

a.  Select the type of test items to use.

b.  Decide the length of the test.

c.  Define the instructional objectives.

d.  Build a table of specification.


10)  Standard Deviation : Variability ; Mean : ________.

a.  discrimination

b.  level of difficulty

c.  correlation

d.  central tendency


11)  Which is implied by a positively skewed score distribution?

a.  The mean, the median and the mode are equal.

b.  Most of the scores are high.

c.  The mode is high.

d.  Most of the scores are low.


12)  Median is the 50th percentile as Q3 is to ________.

a.  45th percentile

b.  70th percentile

c.  75th percentile

d.  25th percentile


13)  Which is true about normal distribution?

a.  The mean, the median and the mode are equal.

b.  Most of the scores are high.

c.  The mode is high.

d.  Most of the scores are low.


14)  Most of the students who took the examination got scores above the mean. What is the graphical representation of the score distribution?

a.  normal curve

b.  mesokurtic

c.  positively skewed

d.  negatively skewed


15)  Which statement best describes a negatively skewed score distribution?

a.  Most examinees got scores above the mean.

b.  The value of median and mode are equal.

c.  The value of mode corresponds to a low score.

d.  The value of median is higher than the value of mode.


16)  Mean is to measure Central Tendency, as ________ is to measure Variability.

a.  Quartile deviation

b.  Quartile

c.  Correlation

d.  Skewness


17)  The distribution of a class with academically poor students is most likely ________.

a.  Normally distributed

b.  Skewed to the left

c.  Skewed to the right

d.  Leptokurtic


18)  Which is correct about the MEDIAN?

a.  It is a measure of variability.

b.  It is the most stable measure of central tendency.

c.  It is the 50th percentile.

d.  It is significantly affected by extreme scores.


19)  Skewed score distribution means.

a.  The scores are normally distributed.

b.  The mean and the median are equal.

c.  The mode, the mean, and the median are equal.

d.  The scores are concentrated more at one end or the other end.


20)  The computed value of r for Math and Science is 0.90. What does this imply?

a.  Math and Science scores are inversely related.

b.  The higher the scores in Math, the higher the scores in Science.

c.  The lower the scores in Science, the lower the scores in Math.

c.  The higher the scores in Science, the higher the scores in Math.


21)  In her conduct of item analysis, Teacher Riel found out that a significantly greater number from the upper group of the class got test item #5 correctly. This means that the test item ________.

a.  has a negative discriminating power

b.  is valid

c.  is easy

d.  has a positive discriminating power


22)  Darwin obtained a NEAT percentile rank of 95. This means that:

a.  They have a zero reference point.

b.  They have scales of equal units.

c.  They indicate an individual's relative standing in a group.

d.  They indicate specific points in the normal curve.


23)  Roxanne obtained a NEAT percentile rank of 95. This means that:

a.  He got a score of 95.

b.  He answered 95 items correctly.

c.  He surpassed in performance 95% of his fellow examinees.

d.  He surpassed in performance 5% of his fellow examinees.


24)  Which objective below is the highest level in Bloom's taxonomy?

a.  Explain how trees receive nutrients.

b.  Explain how a tree functions in relation to the ecosystem.

c.  Rate three different methods of controlling tree growth.

d.  List the parts of a tree.


25)  What will be the most likely distribution if a class is composed of bright students?

a.  Platykurtic

b.  Skewed to the right

c.  Skewed to the left

d.  Leptokurtic


26)  This is a type of statistics that draws conclusions about the sample being studied.

a.  Inferential and corelational

b.  Inferential

c.  Descriptive

d.  Corelational


27)  About how many percent of the cases fall between -2SD and +2SD in the normal distribution curve?

a.  95.44

b.  68.26

c.  99.72

d.  99.85


28)  Which error do teachers commit when they tend to overrate the achievement of students identified by aptitude tests as gifted because they expect achievement and giftedness to go together?

a.  Generosity error

b.  Central tendency error

c.  Severity error

d.  Logical error


29)  Paul's scores in Math quizzes are as follows:  90, 85 70, 65, 99, 78. What is the mean of these scores?

a.  65

b.  99

c.  85.5

d.  81.17


30)  When a significantly greater number from the lower group gets a test item correctly, this implies that the test item ________.

a.  is very valid

b.  is not valid

c.  is not reliable

d.  is highly reliable


31)  On which assumption is portfolio assessment based?

a.  Portfolio assessment is dynamic assessment.

b.  Assessment should stress the reproduction of knowledge.

c.  An individual learner is inadequately characterized by a test score.

d.  An individual learner is adequately characterized by a test score.


32)  What is the mode of the following scores:  90, 85 96, 85, 93, 66, 85, 75, 90, 96, 93, 100?

a.  85

b.  93

c.  93 & 85

d.  85 & 90


33)  A test item has a difficulty index of .81 and a discriminating index of .13. What should the test constructor do?

a.  Retain the item

b.  Make it a bonus item

c.  Revise the item

d.  Reject the item


34)  For mastery learning, which type of testing will be most fit?

a.  Formative testing

b.  Criterion-referenced testing

c.  Aptitude testing

d.  Norm-referenced testing


35)  Which is a characteristic of an imperfect type of matching test?

a.  An item may have no answer at all.

b.  The items in the right and left columns are equal in number.

c.  An answer may be repeated.

d.  There are two or more distracters.


36)  Which is included in item analysis?

a.  Determining the percentage equivalent of the cut-off score.

b.  Identifying the highest score.

c.  Determining the cut-off score.

d.  Determining the effectiveness of distracters.


37)  A positive discrimination index means that ________.

a.  The test item cannot discriminate between the lower and upper groups.

b.  More from the upper group got the item correctly.

c.  More from the lower group got the item correctly.

d.  The test item has low reliability.


38)  When points in the scattergram are spread evenly in all directions, this means that ________.

a.  The correlation between two variables is positive.

b.  The correlation between two variables is low.

c.  The correlation between variables is high.

d.  There is no correlation between two variables.


39)  Which of the following measures is more affected by an extreme score?

a.  Semi-interquartile range

b.  Median

c.  Mode

d.  Mean


40)  Which applies when the mean are equal?

a.  The distribution of the scores is concentrated.

b.  The distribution of the scores is widespread.

c.  The distribution of the scores is identical.

d.  The distribution of the scores are not necessarily identical.


41)  In research, which is another term for independent variable?

a.  Response

b.  Outcome

c.  Criterion

d.  Input


42)  Jennifer's raw score in Math class is 45, which is equal to 96th percentile. What does this mean?

a.  96% of Jennifer's classmates got a score higher than 45.

b.  96% of Jennifer's classmates got a score lower than 45.

c.  Jennifer's score is less than 45% of his classmates.

d.  Jennifer's score is higher than 96% of his classmates.


43)  Which of the following is a norm-referenced statement?

a.  Nikki performed better in spelling than 60% of her classmates.

b.  Nikki was able to spell 90% of the words correctly.

c.  Nikki was able to spell 90% of the words correctly and spelled 35 words out of 50 correctly.

d.  Nikki spelled 35 words out of 50 correctly.


44)  What does a negatively skewed score contribution imply?

a.  The scores congregate on the left side of the normal distribution curve.

b.  The scores are widespread.

c.  The students must be academically poor.

d.  The scores congregate on the right side of the normal distribution curve.


45)  What type of validity  is needed for a test on course objectives and scopes?

a.  Content

b.  Concurrent

c.  Criterion

d.  Construct


46)  Which applies when there are extreme scores?

a.  The median will not be a very reliable measure of central tendency.

b.  The mode will be the most reliable measure of central tendency.

c.  The mean will not be a very reliable measure of central tendency.

d.  There is no reliable measure for central tendency.


47)  The sum of all the scores in a distribution always equals to ________.

a.  the mean times the interval size.

b.  the mean divided by the interval size.

c.  the mean times the N.

d.  the mean divided by N.


48)  The teacher gives an achievement test to his 25 students. The test consists of 50 items. He wants to classify his students' performance based on the test result.

a.  Z-value

b.  Percentile rank

c.  Stanine

d.  Percentage


49)  What measure of position is appropriate when the distribution is skewed?

a.  Mean

b.  Stanine

c.  Z-value

d.  Percentile rank


50)  You give a 100-point test. Three students make a scores of 95, 92 and 91 respectively, while the other 22 students in the class got scores ranging from 33 to 67. The measure of central tendency which is apt to best describe for this group of 25 students is ________.

a.  the mean

b.  the mode

c.  an average of the median and the mode

d.  the median


ANSWERS:


1. C

2. B - The Z-score for Science is 3.67, while the Z-score of Math is 2, and the Z-score of Filipino is -0.42. Using the concept of Z-score, the greater the value of Z-score, the better the performance of a certain student is.

3. C

4. B - The CV of Science is 6.12%. The smaller the value of CV, the closer the scores are. 

5. C - The CV of Filipino is 12.63%. The larger the value of CV, the scores are more widespread, scattered or heterogeneous.

6. B

7. A

8. C

9. C

10. D

11. D

12. C

13. A

14. D

15. A

16. A

17. C

18. C

19. D

20. B - Because r = 0.90 means a high positive correlation.

21. D

22. C

23. C

24. C

25. C

26. B

27. A

28. A

29. D

30. C

31. A

32. A

33. D

34. A

35. A

36. D

37. B

38. D

39. D

40. C

41. D

42. B

43. A

44. D

45. A

46. C

47. C

48. A

49. A

50. A - Because in using the mean, you are going to utilize all the scores of the students. This, using the mean, you can describe best their group performance.

#ProfEd LET Reviewer 

150 Items with Answer Key

1. At the end of periodical examination, Teacher R administered a summative test in Filipino. After scoring the test papers she assigned grades to each test score such as 95, 90, 85, 80 etc. What process did Teacher R use?

 A. Ranking

 B. Computation

 C. Measurement

 D. Evaluation

2. It is the study of man’s pre­history through the buried remains of ancient culture, skeletal remnants of human beings.

 A. Anthropology

 B. Archeology

 C. Ethnology

 D. Ethnography

3. Teacher D claims: "If I have to give reinforcement, it has to be given immediately after the response." Which theory supports Teacher D?

 A. operant conditioning theory

 B. social cognitive theory

 C. cognitive theory

 D. humanist theory

4. Visual imagery helps people store information in their memory more effectively. Which is one teaching implication of this principle?

 A. You will not object when your students daydream in class.

 B. Instruct students to take notes while you lecture.

 C. Encourage your students to imagine the characters and situations when reading a story.

 D. Tell them to read more illustrated comics.

5. Which is essential in the cognitive development of a person according to Vygotsky?

 A. independent thinking

 B. social interaction

 C. individual mental work

 D. scientific thinking

6. The design of the 2002 Basic Education Curriculum (BEC) is based on the principles that the main sources for contemporary basic education are the expert systems of knowledge and the learner's experience in their context. This shows that the BEC is _____ in orientation.

I. constructivist

II. behaviorist

III. essentialist

 A. I and III

 B. III only

 C. I only

 D. I, II, and III

7. Which assessment tool shows evidence of student’s writing skills?

 A. project

 B. portfolio

 C. critiquing sessions

 D. daily journal

8. Ask to do a learning task, Joe hesitates and says "Mahirap. Ayaw ko. 'Di ko kaya!" (It's difficult. I don't like it. I can't do it.) To which problem does the case of the student allude?

I. Unmotivated students

II. Uncaring teachers

III. Extremely difficult learning tasks

IV. incompetent teachers

 A. I and III

 B. I and II

 C. II and III

 D. I, II, and III

9. Teacher S wants to determine immediately the learning difficulties of her students. Which of the following do you expect her to undertake?

 A. Require her students to prepare a portfolio.

 B. Administer an achievement test.

 C. Administer a diagnostic test.

 D. Interview her students directly.

10. NSAT and NEAT results are interpreted against set mastery level. This means that NSAT and NEAT fall under __________.

 A. intelligence test

 B. aptitude test

 C. criterion-referenced test

 D. norm-referenced test

11. The result of the item analysis showed that item no. 4 has a discrimination index of 0.67. What characteristic could be true about this item?

 A. Difficult

 B. Valid

 C. Easy

 D. Average

12. The theme of Vygotsky's socio-cultural theory emphasizes the role of appropriate assistance given by the teacher to accomplish a task. Such help enables the child to move from the zone of actual developmeny to a zone of proximal development. Such assistance is termed _____.

 A. competency technique

 B. scaffolding

 C. active participation

 D. collaboration

13. Which of these activities is not appealing to the bodily-kinesthetic learners?

 A. making math moves

 B. doing simple calisthenics

 C. sketching/illustrating events

 D. joining extramural events

14. Institutions of learning are required to meet the minimum standards for state recognition but are encouraged to set higher standards of quality over and above the minimum through _____ as provided in Educational Act of 1982.

 A. lifelong education 

 B. voluntary accreditation

 C. formal education

 D. academic freedom

15. You intend to assess affective attributes such as capacity to feel, attitudes, and behavior. Which of the following should you establish to ascertain the instrument's validity?

 A. construct

 B. content

 C. criterion-related

 D. face

16. Which will be the most authentic assessment tool for an instructional objective on working with and relating to people?

 A. conducting mock election

 B. home visitation

 C. organizing a community project

 D. writing articles on working and relating to people

17. As an effective classroom manager, what should a teacher do?

I. She uses instructional time wisely.

II. She uses her power to punish students for the sake of discipline.

III. She puts to use the available and appropriate materials.

IV. She manipulates colleagues and  students so she can meet her goals.

 A. I and III

 B. II, III, and IV

 C. I, II, and III

 D. I, II, III, and IV

18. Keeping track of assessment results from one periodic rating to the next is useful in contributing to the development of a _____.

 A. regional plan

 B. annual implementation plan

 C. school improvement plan

 D. division plan

19. In the K­ W­ L technique K stands for what the pupil already knows, W for he wants to know and L for what he _____.

 A. failed to learn

 B. he likes to learn

 C. needs to learn

 D. learned

20. Which is one characteristic of an effective classroom management?

 A. It quickly and unobtrusively redirects misbehavior once it occurs.

 B. It teaches dependence on others for self-control.

 C. It respects cultural norms of a limited group students.

 D. Strategies are simple enough to be used consistently.

21. When curriculum content is fairly distributed in each area of discipline, this means that the curriculum is _____.

 A. sequenced

 B. balanced

 C. integrated

 D. continued

22. Direct instruction is for facts, rules, and actions as indirect instruction is for _____.

 A. hypotheses, verified data, and conclusions

 B. concepts, patterns, and abstractions

 C. concepts, processes, and generalizations

 D. guesses, data, and conclusions

23. The difficulty index of a test item is 1.0. This means that the test is _____.

 A. a quality item

 B. very difficult

 C. very easy

 D. missed by everybody

24. The discrimination index of a test item is -0.35. What does this mean?

 A. More from the upper group got the item correctly.

 B. More from the lower group got the item correctly.

 C. The test is quite reliable.

 D. The test item is valid.

25. His aim of education is individual not a preparation for but participation in the life around the individual.

 A. Froebel

 B. Spencer

 C. Herbart

 D. Pestalozzi

26. The following are characteristics of a child-friendly school except for _____.

 A. exclusive

 B. child-centered

 C. gender-sensitive

 D. nondiscriminating

27. To make the lesson meaningful, systematic and motivating, teachers’ example should be ______.

 A. based on higher ­level skills

 B. interesting aided with illustrations

 C. easy, simple, and understandable

 D. relevant to students’ experience and knowledge

28. Which are the most important concerns about the use of ICT in instruction?

I. Developing appropriate curriculum materials that allow students to construct meaning and develop knowledge throught the use of ICT

II. Devising strategies to meaningfully integrate technology into the curriculum

III. Using pedagogical skills related to technology

IV. Providing teachers with skills for using software applications

 A. I and III

 B. II and III

 C. III and IV

 D. I, II, III, and IV

29. Which of these non-threatening means of assessing learning outcomes?

I. Portfolio

II. Self-evaluation

III. Peer evaluation

IV. Learning journals

 A. I and II

 B. I and III

 C. I, II, and IV

 D. II and III

30. Teacher A's lesson is about the parts of the gumamela. He asked his pupils per group to bring a real flower to study the different parts. After the group work labeling each part, the teacher gave a test. What would be the best type of test he can give?

 A. essay type

 B. matching type

 C. diary

 D. journal

31. Principal A wants her teachers to be constructivist in their teaching orientation. On which assumption/s is the principal's action anchored?

I. Students learn by personally constructing meaning of what is taught.

II. Students construct and reconstruct meanings based on experiences.

III. Students derive meaning from the meaning that teacher gives.

 A. I and III

 B. I only

 C. I and II

 D. II only

32. The norms in a school culture are centered on the _____.

 A. learner

 B. teacher

 C. principal

 D. supervisor

33. The difficulty index of a test item is 1.0. What does this mean?

 A. The test item is very good, so retain it.

 B. The test item is very difficult.

 C. The test item is extremely easy.

 D. The test item is not valid.

34. Someone wrote: "Environment relates to the profound relationship between matter, nature, and society, and in such a context, ICTs bring new ways of living in a more interconnected society, all of which reduces our dependency on matter and affects our relationship with nature." What does this convey?

 A. Environment and ICT are poles apart.

 B. ICT impacts on environment.

 C. Environment affects ICT.

 D. ICT brings us away from an interconnected society.

35. Educational institutions’ effort of developing work skills inside the school are aimed at _____.

 A. developing moral character

 B. inculcating love of country 

C. teaching the duties of citizenship

 D. developing vocational efficiency

36. Teacher B discovered that her pupils are weak in comprehension. To further determine in which particular skills her pupils are weak which test should Teacher B give?

 A. Aptitude Test

 B. Placement Test

 C. Diagnostic Test

 D. Standardized Test

37. That the quality of Philippine education is declining was the result of a study by EDCOM which recommended to _____ teachers and teaching.

 A. regulate

 B. progessionalize

 C. strengthen

 D. improve

38. As provided in the Republic Act #4670, every teacher shall enjoy equitable safeguards at each stage of any disciplinary procedures and shall the following except:

 A. the right to be informed in writing, of the charges

 B. the right to full access to the evidence in the case

 C. the right to appeal to clearly designated authorities.

 D. All of these

39. The subject matter or content to be learned must be within the time allowed, resources available, expertise of the teacher, and nature of learners. What criterion is addressed?

 A. validity

 B. significance

 C. interest

 D. feasibility

40. A child was shown an amount of water in a glass. The teacher poured the whole amount to a much taller and narrower glass and marked this glass as

 A. The same amount was poured in a shorter and wider glass, marked glass

 B. When asked which has more water, the child's answer was "Glass A". In what stage of cognitive development is the child and what is this ability called? A. concrete operational stage; conservation B. formal operational stage; deductive reasoning

 C. sensorimotor stage; symbolic functions

 D. pre-operational stage; centration

41. In a multiple choice test item with four options and out of 50 examinees, which was the least effective distracter?

 A. the option that was chosen by 13 examinees

 B. the option that was chosen by 2 examinees

 C. the option that was chosen by 30 examinees

 D. the correct answer that was chosen by 5 examinees

42. What is an alternative assessment tool for teaching and learning consisting of a collection of work/artifacts finished or in-progress accomplished by the targeted clientele?

 A. rubric

 B. achievement test

 C. evaluation instrument

 D. portfolio

43. Societal change requires continually deep-seated questions about "good" living. Which of these did Socrates recognize as the greatest of human virtues?

 A. moral wisdom

 B. fair justice

 C. courage

 D. piety

44. Among the following curriculum stakeholders, who has the most responsibility in curriculum implementation?

 A. the learners

 B. the school heads

 C. the teachers

 D. the parents

45. Study this group of tests which was administered with the following results, then answer the question.

 SubjectMathPhysicsEnglishMean564180SD10916Student′sScore4331109

In which subject(s) did Ronnel perform most poorly in relation to the group’s performance?

 A. English

 B. English and Math

 C. Math

 D. Physics

46. You like to show a close representation of the size and shape of the earth and its location in the entire solar system. What is the best instructional aid?

 A. picture

 B. model

 C. realia

 D. film

47. Which of the following techniques of curriculum implementation is fit to the objective of developing cooperative learning and social interaction?

 A. buzz session

 B. graded recitation

 C. individual reporting

 D. lecture

48. To determine your pupil's entry knowledge and skills, which should you employ?

 A. interview

 B. focus group discussion

 C. post-test

 D. pre-test

49. We encounter people whose prayer goes like this: “O God, if there is a God; save my soul, if I have a soul” From whom is this prayer?

 A. Stoic

 B. Empiricist

 C. Agnostic

 D. Skeptic

50. What is the implication of using a method that focuses on the why rather than the how?

 A. There is best method

 B. Typical one will be good for any subject

 C. These methods should be standardized for different subjects.

 D. Teaching methods should favor inquiry and problem solving.

51. Which software can you predict changes in weather pattern and or trends in the population of endangered species?

 A. word processing

 B. spreadsheet

 C. desktop publishing

 D. database

52. You would like to assess students' ability to write a portfolio. What type of test will determine their ability to organize ideas and think critically?

 A. long test

 B. essay test

 C. formative test

 D. summative test

53. Teacher F asks one student, "Rachel, can you summarize what we have just read? Remember, the title of this section of the chapter." This is an example of a teacher's effort at _____.

 A. scaffolding

 B. inspiring

 C. directing

 D. giving feedback

54. Cooperative learning approach makes use of a classroom organization where students work in teams to help each other learn. What mode of grouping can facilitate the skill and values desired?

 A. large group

 B. homogeneous

 C. heterogeneous

 D. Wear multicolored dress to catch the students' attention.

55. To solve moral ambiguity among us Filipinos, we must _____.

 A. excuse ourselves whenever we do wrong

 B. blame our government for not doing anything about it

 C. be aware and responsible about the problem

 D. be comfortable with the present state affairs

56. In their desire to make schools perform, the DepEd then published the ranking of schools in NAT results nationwide. As an effect of this practice, what did schools tend to do?

I. Taught at the expense of NAT

II. Conducted review classes for NAT at the expense of teaching

III. Practiced the so-called teaching to the test

 A. II and III

 B. II only

 C. I and III

 D. III only

57. Which one appropriately describes your lesson if you use the cognitive approach?

 A. promotes "find out for yourself" approach

 B. lecture-dominated

 C. rote learning dominated

 D. highly directed teaching

58. Which apply/applies to extrinsically motivated learners?

I. Tend to process information superficially

II. Tend to be content with meeting minimum requirements

III. Achieve at high level

 A. I and II

 B. II only

 C. I and III

 D. I only

59. To make our children become like "little scientists", which of the following methods should we employ more often?

I. Inquiry

II. Problem-solving

III. Laboratory

 A. II and III

 B. I and II

 C. I, II, and III

 D. I and III

60. Teacher B is a teacher of English as a Second Language. She uses vocabulary cards, fill-in-the-blank sentences, dictation and writing exercises in teaching a lesson about grocery shopping. Based on this information, which of the following is a valid conclusion?

 A. The teacher is reinforcing learning by giving the same information in, a variety of methods.

 B. The teacher is applying Bloom's hierarchy of cognitive learning.

 C. The teacher wants to do less talk.

 D. The teacher is emphasizing listening and speaking skills.

61. A mother gives her son his favorite snack every time the boy cleans up his room. Afterwards, the boy cleans his room everyday in anticipation of the snack. Which theory explains this?

 A. operant conditioning

 B. social learning theory

 C. associative learning

 D. Pavlovian conditioning

62. What is the primary fundamental question in examining a curriculum?

 A. What educational experiences can be provided that are likely to attain these purposes?

 B. What educational purposes should the school seek to attain?

 C. How can these educational experiences be effectively organized?

 D. How can we determine whether these purposes are attained or not?

63. To elicit more student’s response, Teacher G made use of covert responses. Which one did she NOT do?

 A. She had the students write their response privately.

 B. She showed the correct answers on the overhead after the students have written their responses.

 C. She had the students write their responses privately then called each of them.

 D. She refrained from judging on the student's responses.

64. Which is the ideal stage of moral development?

 A. social contract

 B. universal ethical principle

 C. law and order

 D. good boy/ good girl

65. There is a statement that says, "No amount of good instruction will come out without good classroom management." Which of the following best explains this statement?

 A. Classroom management is important to effect good instruction.

 B. There must be classroom management for instruction to yield good outcomes/ results.

 C. Classroom management means good instruction.

 D. Good instruction is equal to effective classroom management.

66. What does a Table of Specification establish? 

A. construct validity

 B. content related validity and criterion reference

 C. content validity and construct validity

 D. content validity and content related validity

67. A person who had painful experiences at the dentist's office may become fearful at the mere sight of the dentist's office. Which theory can explain this?

 A. generalization

 B. classical conditioning

 C. operant conditioning

 D. attribution theory

68. Which one can help student develop the habit of critical thinking?

 A. Asking low level questions

 B. A willingness to suspend judgment until sufficient evidence is represented

 C. Asking convergent questions

 D. Blind obedience to authority

69. Multiple intelligences can be used to explain children's reading performance. Which group tends to be good readers?

 A. linguistically intelligent group

 B. spatially intelligent group

 C. existentially intelligent group

 D. kinesthetically intelligent group

70. Teachers should avoid _____ in assigning student performance-based ratings.

 A. arbitrarines and bias

 B. unnecessary deductions

 C. partiality and calculation

 D. unnecessary evaluation

71. Which of the following is NOT a guidance role of the classroom teacher?

 A. Psychological Test Administrator

 B. Listener­Adviser

 C. Human Potential Discoverer

 D. Total Development Facilitator

72. Watson applied classical conditioning in his experiments and the results showed that behavior is learned through stimulus-response associations, specifically the development of emotional responses to certain stimuli. This helps is in _____.

 A. interpreting reflexes as emotions

 B. understanding fears, phobias, and love

 C. connecting observable behavior to stimulus

 D. understanding the role of overt behavior

73. Who claimed that children are natural learners and therefore, must be taught in natural settings?

 A. Piaget

 B. Froebel

 C. Montessori

 D. Kohlberg

74. the benefit of "reading aloud" is that children learn _____.

 A.  new vocabulary in meaningful contexts

 B. to value the presence of their friends as they read together

 C. to make predictions by examining pictures and listening for clues

 D. to use their imaginations to explore new ideas as they listen to books

75. What does a negatively skewed score contribution imply?

 A. The scores congregate on the left side of the normal contribution curve.

 B. The scores are widespread.

 C. The students must be academically poor.

 D. The score congregate on the right side of the normal contribution curve.

76. A high school graduate was refused admission to a university on the grounds that he failed the admission test. The student insisted that he had the right to be admitted and the act of the univeristy was a violation of his right to education. Was the student correct?

 A. No, the university may refuse the student in its exercise of academic freedom.

 B. Yes, education is everyone's right.

 C. Yes, especially if he belongs to the Indigenous Peoples' group.

 D. No, if the university is exclusively for girls.

77. Which of the following is a correct statement on service contracting scheme?

 A. It increases access to education.

 B. It works against quality education.

 C. It discriminates against private schools.

 D. it is not cost-effective.

78. Student M obtained an NSAT percentile rank of 80. This indicates that _____.

 A. he surpassed in performance 80% of her fellow examinees

 B. he got a score of 80

 C. he surpassed in performance 20% of her fellow examinees

 D. he answered 80 items correctly

79. Teachers are encouraged to make use of authentic assessment. Which goes with authentic assessment?

 A. unrealistic performances

 B. de-contextualized drills

 C. real world application of lessons learned

 D. answering high multiple choice test items

80. Global students learn with short bursts of energy. To maintain concentration they require ______.

 A. frequent reminder that they need to concentrate

 B. frequent and intermittent breaks

 C. short and easy reading materials

 D. music while studying

81. A child was punished for cheating in an exam. For sure the child wont cheat again in short span of time, but this does not guarantee that the child won’t cheat ever again. Based on Thorndike’s theory on punishment and learning, this shows that _____.

 A. punishment strengthens a response

 B. punishment doesn’t remove a response

 C. punishment removes response

 D. punishment weakens a response

82. Faith, hope, and love are values now and forever whether they will be valued by people or not. Upon what philosophy is this anchored?

 A. Idealism

 B. Existentialism

 C. Realism

 D. Pragmatism

83. In a grade distribution, what does the normal curve mean?

 A. a large number of students receiving low grades and very few students with high marks

 B. a large number of more or less average students and very few students receiving low and high grades

 C. a large number of students with high grades and very few with low grades

 D. all of the students have average grades

84. Lecturer C narrates: "I observe that when there is an English-speaking foreigner in class, more often than not, his classmates perceive him to be superior." To which Filipino trait does this point?

 A. hospitality

 B. friendliness

 C. colonial mentality

 D. lack of confidence

85. Which violates this brain-based principle of teaching-learning: "Each child's brain is unique and vastly different from one another."

 A. giving ample opportunity for a pupil to explore even if the class creates "noise"

 B. making a left-handed pupil write with his right hand as it is better this way

 C. allowing open dialogue among students

 D. employing MI teaching approaches

86. The child fainted in your class because she has not eaten her breakfast. What is the best thing for you to do in this situation?

 A. Ignore the situation

 B. Comfort the child

 C. Give the child food

 D. Call the parent

87. Which one should teacher AVOID to produce an environment conducive for learning?

 A. Tests

 B. Seat plan

 C. Individual competition

 D. Games

88. You are convinced that whenever a student performs a desired behavior, provide reinforcement and soon the student learns to perform the behavior on her own. On which principle is your conviction based?

 A. cognitivism

 B. behaviorism

 C. constructivism

 D. environmentalism

89. “Men are built not born.” This quotation by John Watson states that _____.

 A. the ineffectiveness of training on a person’s development

 B. the effect of environmental stimulation on a person’s development

 C. the absence of genetic influence on a person’s development

 D. the effect of heredity

90. Which order follows the basic rule in framing interaction?

 A. call on a student, pause, ask the question

 B. ask the question, call on a student, pause

 C. ask the question, pause, call on a student

 D. call on a student, ask the question, pause

91. At the preoperational stage of Piaget's cognitive development theory, the child can see only his point of view and assumes that everyone also has the same view as his. What is this tendency called?

 A. transductive reasoning

 B. animism

 C. egocentrism

 D. conservatism

92. "Vox Populi Est Supreme Lex" is a Latin expression that means what?

 A. The Supreme Being is God

 B. No one is above the law

 C. The voice of the people is the supreme law

 D. it is the popular choice

93. Which is not among the major targets of the Child-Friendly School System (CFSS)?

 A. All school children are friendly.

 B. All children 6-12 years old are enrolled in elementary schools.

 C. All children complete their elementary education within six years.

 D. All Grade 6 students pass the division, regional, and national tests.

94. Thorndike's law of effect states that a connection between stimulus and response is strengthened when the consequence is _____.

 A. repeated

 B. negative

 C. pleasurable

 D. positive

95. Which is the most reliable tool of seeing the development in your pupils' ability to write?

 A. portfolio asssessment

 B. scoring rubric

 C. interview of pupils

 D. self-assessment

96. The Philippine constitution directs the teaching of religion in public schools on the following conditions except for?

 A. option is expressed in writing

 B. without cost to the government

 C. with cost shouldered by the parents or guardians

 D. given only at the option of parents or guardians

97. These are also known as “combination classes” organized in barrios/barangays where the required number of pupils of the same grade levels has not met the required number to make a separate class thus the teacher apportions class time for instruction to every grade level within class.  These are ______.

 A. extension classes

 B. heterogeneous classes

 C. multi­grade classes

 D. homogeneous classes

98. After reading and paraphrasing R. Frost's Stopping by the Woods on a Snowy Evening, Teacher M asked the class to share any insight derived from this poem. On which assumption about the learner is Teacher M's act of asking the class to share their insight based?

 A. Learners are producers of knowledge, not only passive recipients of information.

 B. Learners are meant to interact with one another.

 C. Learners are like empty receptacles waiting to be filled up.

 D. Learners have multiple intelligence and varied learning styles.

99. "Do not cheat. Cheating does not pay. If you do, you cheat yourself." says the voiceless voice from within you. In the context of Freud's theory, which is at work

 A. id alone

 B. superego alone

 C. ego alone

 D. Id and ego interaction

100. Which of the following assessment tools would you recommend if one should adhere to constructivist theory of learning?

I. Constructed response test

II. Performance test

III. Checklist of a motor screening test

IV. Observation test 

 A. I and II

 B. II and III

 C. I, II, and III

 D. I, II, and IV

101. The test in English and Mathematics showed poor results in comprehension and problem-solving questions. How may the data be used for better learners' performance?

 A. Use context clues in vocabulary building.

 B. Give more exercises/situations on comprehension questions.

 C. Determine weakness in grammatical structures.

 D. Involve parents in guiding learners' developing good study habits.

102. Which of the following may be the assumption/s of behaviorists?

I. The mind of a newborn child is a blank slate.

II. All behaviors are determined by environmental events.

III. The child has a certain degree of freedom not to allow himself to be shaped by his environment.

 A. III only

 B. I and II

 C. I and III

 D. II only

103. _____ supports equitable access but on the other hand, quality might be compromised. A. Open admission B. School accreditation C. Deregulated tuition fee D. Selective retention

104. If you want your students to develop reading comprehension and learning strategies which one should you employ?

 A. reciprocal teaching

 B. cooperative learning

 C. peer tutoring

 D. mastery learning

105. What statement is FALSE with reference to Article VIII “The Teacher and the Learners” of the Code of Ethics of Professional Teachers?

 A. A teacher shall not accept favors or gifts from learners, their parents, or others in their behalf in exchange for requested concessions, especially if undeserved,

 B. A teacher shall not inflict corporal punishment or offending learners nor make deductions from their scholastic ratings as a punishment for acts which are clearly not manifestation of poor scholarship.

 C. In a situation where mutual attraction and subsequent love develop between teacher and the learner, the teacher shall exercise utmost professional discretion to avoid scandal, gossip, and preferential treatment of the learner.

 D. A teacher shall maintain at all ties a dignified personality, which could serve as model worthy of emulation by learners, peers and others.

106. In the light of the modern concept of teaching, which is a characteristic of effective teaching?

 A. pouring information to the learners

 B. allowing learners to learn on their own

 C. developing abilities to address the future

 D. removing the physical presence of the teacher

107. One facet of understanding, an evidence of learning is perspective. Which is an indicator of perspective?

 A. A bright student refuses to consider that there is another correct solution to the problem apart from hers.

 B. A student explains the arguments for and against the acquittal of  Hubert Webb and group.

 C. A teacher cannot accept opinions different from hers.

 D. A mother cannot understand why her child's performance is below par.

108. Which among the indicators could be most useful for assessing quality of schooling?

 A. participation rate

 B. cohort survival rate

 C. net enrollment rate

 D. drop-out rate

109. The discrimination index of a test item is +0.48. What does this mean?

 A. An equal number from the lower and upper group got the item correctly.

 B. More from the upper group got the item wrongly.

 C. More from the lower group got the item correctly.

 D. More from the upper group got the item correctly.

110. Which of the following is NOT an example of a teacher’s nonverbal communication?

 A. eye contact

 B. gestures

 C. pauses

 D. voice

111. Which is a valid assessment tool if you want to find out how well your students can speak extemporaneously?

 A. performance test in extemporaneous speaking

 B. written quiz on how to deliver extemporaneous speech

 C. display of speeches delivered

 D. writing speeches

112. Professionalization of teachers and teaching as promulgated in Presidential Decree No. 1006, defines teaching as profession concerned with classroom institution ______.

 A. by teachers on full­time basis

 B. at the tertiary level in both public and private institutions

 C. at the elementary and secondary levels in both public and private schools

 D. by teachers of permanent status

113. Teacher U asked her pupils to create a story out of the given pictures. Which projective technique did Teacher U use?

 A. Rorschach test

 B. narrative

 C. thematic apperception test

 D. reflective

114. The Code of Ethics for Professional Teachers stipulates that educational institutions shall offer quality education for all Filipino citizens. How is quality education defined in R.A. 9155?

 A. Relevance and excellence of education are emphasized to meet the needs and aspirations of an individual and society.

 B. All school-aged children should be provided free and compulsory education.

 C. Children with special needs should be mainstreamed with regular classes in the public schools.

 D. Public and private basic education schools should provide relevant education.

115. Teacher A discovered that his pupils are very good in dramatizing. Which tool must have helped him discover his pupils’ strength?

 A. Portfolio assessment

 B. Performance test

 C. Journal entry

 D. Paper-and-pencil test

116. Which appropriate teaching practice flows from this research finding on the brain: The brain's emotional center is tied into its ability to learn.

 A. Create a learning environment that encourages students to explore their feelings and ideas freely.

 B. Come up with highly competitive games where winners will feel happy.

 C. Tell students to participate in class activities or else won't receive plus points in class recitation. D. To establish discipline, be judgmental in attitude.

117. Your percentile rank in class is 60%. What does this mean?

 A. You got 40% of the test items wrongly.

 B. You scored less than 60% of the class.

 C. You got 60% of the test items correctly.

 D. You scored better than 60% of the class.

118. Which test item is in the highest level of Bloom's taxonomy of objectives?

 A. Explain how a tree functions in relation to the ecosystem.

 B. Explain how trees receive nutrients.

 C. Rate three different methods of controlling tree growth.

 D. Write a paragraph that observes coherence, unity, and variety.

119. In qualitative social and behavioral studies, "the investigator is a part of the study." What are implied in this statement?

I. The researcher processes and analyzes the data himself.

II Data interpretation depends on the orientation of the researcher.

III. The investigator is the only source of information.

IV. Data gathering may be done by others but the analysis is done by the researcher.

 A. I and IV

 B. II and III

 C. I, II, and IV

 D. I, II, III, and IV

120. The observable manifestation of student's feelings, thoughts, or attitude are summed up as behavior. Every high school teacher is expected to contribute to the assessment of the student's behavior but the grade is reflected in _____.

 A. Good Manners and Right Conduct

 B. Values Education

 C. co-curricular activities

 D. curricular activities

121. Which must be present for self-evaluation to succeed?

 A. consensus between teacher and student regarding evaluation results

 B. teacher's approval of self-evaluation results

 C. teacher's monitoring of self-evaluation process

 D. student's intrinsic motivation to learn

122. The instructions for a test are made simple, clear and concise. This is part of which of the following characteristics of a good test?

 A. objectivity

 B. economy

 C. administrability

 D. scorability

123. What is the mean of this score distribution: 4, 5, 6, 7, 8, 9, 10?

 A. 7

 B. 6

 C. 8.5

 D. 7.5

124. The Early Childhood Care and Development Act provides for the promotion of the rights of children for survival and development. Which of the following statements is not among its objectives?

 A. Enhance the role of parents as the primary caregivers and educators of their children form birth onward.

 B. Facilitate a smooth transition from care and education provided at home to community or school-based setting and primary schools.

 C. Assist the LGUs in their endeavor to prepare the child for adulthood.

 D. Enhance the physical, social, emotional, cognitive, psychological, spiritual, and language development of young children.

125. Which of the following represents a minuscule curriculum?

 A. textbook that learners use

 B. lesson plan that teachers prepare

 C. reference materials that supplement the text

 D. lecture notes of the teacher

126. Bernadette enjoyed the roller coaster when they went to Enchanted Kingdom. Just at the sight of a roller coaster, she gets excited. Which theory explains Bernadette's behavior?

 A. operant conditioning

 B. attribution theory

 C. Pavlovian conditioning

 D. social learning theory

127. What objectives do effective leaders foster?

I. Attainable

II.Measurable

III.Results-oriented

IV. Specific

V. Time-bound

 A. II, III, and IV

 B. I, II, III, IV, and V

 C. I, IV, and V

 D. I, II, III, and IV

128. Thomasites are _____.

 A. the soldiers who doubted the success of the public educational system to be set in the Philippines

 B. the first American teacher recruits to help establish the public educational system in the Philippines

 C. the first religious group who came to the Philippines on board the US transport named Thomas.

 D. the devotees of St. Thomas Aquinas who came to evangelize

129. Which is an underlying assumption of the social cognitive theory?

 A. People are social by nature.

 B. People learn by observing others.

 C. People learn by trial-and-error.

 D. People learn by association.

130. Researchers found that when a child is engaged in a learning experience, a number of areas of the brain are simultaneously activated. What is an application of this in the teaching-learning process?

I. Make use of field trips, guest speakers

II. Do multicultural units of study

III. Stick to the "left brain and right brain" approach

 A. II only

 B. I only

 C. I and III

 D. I and II

131. Here is a score distribution: 98, 93, 93, 93, 90, 88, 87, 85, 85, 85, 70, 51, 34, 34, 34, 20, 18, 15, 12, 9, 8, 6, 3, 1. Which is the range?

 A. 93

 B. 85

 C. 97

 D. between 51 and 34

132. Which tool should a teacher use if she wants to locate areas which are adversely affecting the performance of a significant number of students?

 A. problem checklist

 B. self-report technique

 C. autobiography

 D. cumulative record

133. Which activity works best with self-expressive people?

 A. metaphors

 B. kinesthetic activities

 C. inquiry

 D. independent study

134. centralization : Education Act of 1901 :: decentralization : _____

 A. R.A. 9155

 B. R.A. 9293

 C. R.A. 7836

 D. R.A. 7722

135. Piagetian tasks state that thinking becomes more logical and abstract as children reach the formal operations stage. What is an educational implication of this finding?

 A. Expect hypothetical reasoning from learners between 12 to 15 years of age.

 B. Engage children in analogical reasoning as early as preschool to train them for HOTS.

 C. Learners who are not capable of logical reasoning from ages 8 to 11 lag behind in their cognitive development.

 D. Let children be children.

136. This embodies the teacher’s duties and responsibilities as well as proper behavior in performing them.

 A. Code of Ethics for Teachers

 B. Magna Carta for Public School Teachers

 C. Bill of Rights

 D. Philippine Constitution of 1987

137. The class was asked to share their insights about the poem. The ability to come up with an insight stems from the ability to _____.

 A. comprehend the subject that is being studied

 B. analyze the parts of a whole

 C. evaluate the worthiness of a thing

 D. relate and organize things and ideas

138. Which is not a characteristic of authentic assessment?

 A. focused on lifelike, meaningful, relevant types of student learning

 B. offers opportunities to study problem intensively

 C. easy to complete

 D. fruitful in terms of genuine learning

139. Schools should develop in the students the ability to adapt to a changing world. This is adhereance to the philosophy of _____.

 A. essentialism

 B. perennialism

 C. progressivism

 D. reconstructionism

140. The teacher’s first task in the selection of media in teaching is to determine the ______.

 A. choice of the students

 B. availability of the media

 C. objectives of the lesson

 D. technique to be used

141. With which goals of educational institutions as provided for by the Constitution is the development of work skills aligned?

 A. To develop moral character

 B. To teach the duties of citizenship

 C. To inculcate love of country

 D. To develop vocational efficiency

142. Median is to point as standard deviation is to _____.

 A. area

 B. volume

 C. distance

 D. square

143. Who is the forerunner of the presence of the Language Acquisition Device?

 A. Watson

 B. Chomsky

 C. Gardner

 D. Piaget

144. Which of the following statements best describes metacognition as a strategy for curriculum augmentation?

 A. It is learning how to learn and thinking about how one thinks.

 B. It is learning strategies for success.

 C. It is learning through interaction with the environment.

 D. It is learning through computer-aided instruction.

145. Which psychological theory states that the mind insists on finding patterns in things that contribute to the development of insight?

 A. Piaget's psychology

 B. Kohlberg's psychology

 C. Gestalt psychology

 D. Bruner's psychology

146. Which is not a characteristic of an analytic learner?

 A. learns whole to part

 B. enjoys memorizing

 C. works on details

 D. sequences objects in order

147. The concepts of trust vs. maturity, autonomy vs. self-doubt, and initiative vs. guilt are most closely related with the works of __________.

 A. Erikson

 B. Piaget

 C. Freud

 D. Jung

148. Ask to do a learning task, Joe hesitates and says "Mahirap. Ayaw ko. 'Di ko kaya!" (It's difficult. I don't like it. I can't do it.) Is it possible to motivate this type of student?

 A. Yes, he can do something with his ability.

 B. Yes, he can change the nature of the job.

 C. No, it is impossible to motivate a student who himself is not motivated.

 D. No, motivation is totally dependent on the student. No person outside him can influence him.

149. As of the Republic Act 7836 the licensure exam for teachers is with the _____.

 A. Commission on Higher Education

 B. Professional Regulation Commission

 C. Department of Education

 D. Civil Service Commission

150. Filipinization is violated if _____.

 A. an educational institution is owned by a corporation of which 40% of the capital is owned by Filipino Citizens

 B. an educational institution owned by a religious order

 C. an American serving as president of the educational institution

 D. an educational institution owned by a charitable institution

Answer Key

1. B

2. B

3. A

4. C

5. A

6. A

7. D

8. A

9. C

10. C

11. D

12. B

13. C

14. B

15. D

16. C

17. A

18. C

19. D

20. A

21. B

22. C

23. C

24. B

25. A

26. A

27. D

28. D

29. A

30. B

31. C

32. A

33. C

34. B

35. D

36. C

37. B

38. D

39. D

40. A

41. B

42. D

43. A

44. C

45. A

46. B

47. A

48. D

49. D

50. D

51. B

52. B

53. A

54. C

55. C

56. A

57. A

58. A

59. C

60. A

61. A

62. B

63. B

64. B

65. B

66. C

67. B

68. B

69. A

70. A

71. A

72. B

73. C

74. B

75. D

76. A

77. A

78. A

79. C

80. B

81. B

82. A

83. B

84. C

85. B

86. C

87. C

88. B

89. B

90. C

91. C

92. C

93. D

94. D

95. A

96. B

97. C

98. A

99. B

100. A

101. B

102. B

103. A

104. A

105. D

106. B

107. B

108. B

109. D

110. D

111. A

112. C

113. C

114. A

115. B

116. A

117. D

118. D

119. A

120. B

121. D

122. C

123. A

124. C

125. B

126. C

127. B

128. C

129. B

130. D

131. C

132. A

133. B

134. A

135. A

136. A

137. D

138. C

139. D

140. C

141. D

142. B

143. D

144. A

145. C

146. A

147. A

148. A

149. B

150. C

--------------

PROFESSIONAL EDUCATION

Set B with ANSWER KEY

100items

1. Which assumption underlies the teacher's use of performance objectives?

A. Not every form of learning is observable.

B. Performance objectives assure the learrier of learning.

C. Learning is defined as a change in the learner's observable performance.

D. The success of learner is based on teacher performance.

2. In the parlance of test construction what does TOS mean?

A. Table of Specifics

B. Table of Specifications

C. Table of Specific Test Items

D. Team of Specifications

3. A student passes a research report poorly written but ornately presented in a folder to make up for the poor quality of the book report content. Which Filipino trait does this practice prove? Emphasis on __________.

A. art over academics

B. substance overporma

C. art over science

D. pormaover substance

4. In a criterion-referenced testing, what must you do to ensure that your test is fair?

A. Make all of the questions true or false.

B. Ask each student to contribute one question.

C. Make twenty questions but ask the students to answer only ten of their choice.

D. Use the objectives for the units as guide in your test construction.

5. Which does Noam Chomsky, assert about language learning for children?

I. Young children learn and apply grammatical rules and vocabulary as they are exposed to them.

II. Begin formal teaching of grammatical rules to children as early as possible.

III. Do not require initial formal language teaching for children.

A. I and III

B. II only

C. I only

D. I and II

6. Which Filipino trait works against the shift in teacher's role from teacher as a fountain of information to teacher as facilitator?

A. Authoritativeness

B. Authoritarianism

C. Hiya

D. Pakikisama

7. If a teacher plans a constructivist lesson, what will he most likely do? Plan how he can

A. do evaluate his students' work

B. do reciprocal teaching

C. lecture to his students

D. engage his students in convergent thinking

8. Who among the following needs less verbal counseling but needs more concrete and operational forms of assistance? The child who __________.

A. has mental retardation

B. has attention-deficit disorder

C. has learning disability

D. has conduct disorder

9. How would you select the most fit in government positions? Applying Confucius teachings, which would be the answer?

A. By course accreditation of an accrediting body

B. By merit system and course accreditation

C. By merit system

D. By government examinations

10. Which types of play is most characteristic of a four to six-year old child?

A. Solitary and onlooker plays

B. Associative and coooperative plays

C. Associative and onlooker plays

D. Cooperative and solitary plays

11. A teacher's summary of a lesson serves the following functions, EXCEPT

A. it links the parts of the lesson

B. lt brings together the information that has been discussed

C. it makes provisions for full participation of students.

D. it clinches the basic ideas or concepts of the lesson.

12. All of the following describe the development of children aged eleven to thirteen EXCEPT __________.

A. they shift from impulsivity to adaptive ability

B. sex differences in IQ becomes more evident

C. they exhibit increase objectivity in thinking

D. they show abstract thinking and judgement

13. Teacher T taught a lesson denoting ownership by means of possessives. He first introduced the rule, then gave examples, followed by class exercises, then back to the rule before he moved into the second rule. Which presenting technique did he use?

A. Combinatorial

B. Comparative

C. Part-whole

D. Sequence

14. "In the light of the facts presented, what is most likely to happen when ... ?" is a sample thought question on

A. inferring

B. generalizing

C. synthesizing

D. justifying

15. Teacher E discussed how electricity flows through wires and what generates the electric charge. Then she gave the students wires, bulbs, switches, and dry cells and told the class to create a circuit that will increase the brightness of each bulb. Which one best describes the approach used?

A. It used a taxonomy of basic thinking skills

B. It was contructivist

C. It helped students understand scientific methodolgy

D. It used cooperative learning

16. Teacher B uses the direct instruction strategy. Which sequence of steps will she follow?

I. Independent practice

II. Feedback and correctiveness

III. Guided student practice

IV. Presenting and structuring

V. Reviewing the previous day's work

A. V-II-IV-III-I

B. III-II-IV-I-V

C. V-lV-III-II-I

D. I-V-II-III-IV

17. Which are direct measures of competence?

A. Personality tests

B. Performance tests

C. Paper-and-pencil tests

D. Standardized test

18. Under which program were students who were not accommodated in public elementary and secondary schools because of lack of classroom, teachers, and instructional materials, were enrolled in private schools in their respective communities at the government's expense?

A. Government Assistance Program

B. Study Now-Pay Later

C. Educational Service Contract System

D. National Scholarship Program

19. Which activity should a teacher have more for his students if he wants them to develop logical-mathematical thinking?

A. Problem solving

B. Choral reading

C. Drama

D. Storytelling

20. An effective classroom manager uses low-profile classroom control. Which is a low-profile classroom technique?

A. Note to parents

B. After-school detention

C. Withdrawal of privileges

D. Raising the pitch of the voice

21. Your teacher is of the opinion that the world and everything in it are ever changing and so teaches you the skill to cope with change. What is his governing philosophy?

A. Idealism

B. Existentalism

C. Experimentalism

D. Realism

22. To come closer to the truth we need togo back to the things themselves.This is the advice of the

A. behaviorists

B. phenomenologists

C. idealists

D. pragmatists

23. Test norms are established in order to have a basis for __________.

A. establishing learning goals

B. interpreting test results

C. computing grades

D. identifying pupils' difficulties

24. A stitch on time saves nine, so goes the adage.. Applied to classroom management, this means that we __________

A. may not occupy ourselves with disruptions which are worth ignoring because they are minor

B. must be reactive in our approach to discipline

C. have to Jesolve minor disruptions before they are out of control

D. may apply 9 rules out of 10 consistently

25. Which criterion should guide a teacher in the choice of instructional devices?

A. Attractiveness

B. Cost

C. Novelty

D. Appropriateness

26. Based on Piaget's theory, what should a teacher provide for children in the sensimotor stage?

A. Games and other physical activities to develop motor skill.

B. Learning activities that involve problems of classification and ordering.

C. Activities for hypothesis formulation.

D. Stimulating environment with ample objects to play with.

27. Who among the following puts more emphasis on core requirements, longer school day, longer academic year and more challenging textbooks?

A. Perennialist

B. Essentialist

C. Progressivist

D. Existentialist

28. The search for related literature by accessing several databases by the use of a telephone line to connect a computer library with other computers that have database is termed __________.

A. compact disc search

B. manual search

C. on-line search

D. computer search

29. With synthesizing skills in mind, which has the highest diagnostic value?

A. Essay test

B. Performance test

C. Completion test

D. Multiple choice test

30. Based on Piaget's theory, what should a teacher provide for children in the concrete operational stage?

A. Activities for hypothesis formulation.

B. Learning activities that involve problems of classification and ordering.

C. Games and other physical activities to develop motor skills.

D. Stimulating environment with ample objects to play with.

31. Read the following then answer the question:

TEACHER: IN WHAT WAYS OTHER THAN THE PERIODIC TABLE MIGHT WE PREDICT THE UNDISCOVERED ELEMENTS?

BOBBY: WE COULD GO TOTHE MOON AND SEE IF THERE ARE SOME ELEMENTS THERE WE DON'T HAVE.

BETTY: WE COULD DIG DOWN TO THE CENTER OF THE EARTH AND SEE IF WE FIND ANY OF THE MISSING ELEMENTS.

RICKY: WE COULD STUDY DEBRIS FROM THE METEORITES IF WE CAN FIND ANY.

TEACHER: THOSE ARE ALL GOOD ANSWERS BUT WHAT IF THOSE, EXCURSIONS TO THE MOON, TO THE CENTER OF THE EARTH, OR TO FIND METEORITES WERE TOO COSTLY AND TIME CONSUMING? HOW MIGHT WE USE THE ELEMENTS WE ALREADY HAVE HERE ON EARTH TO FIND SOME NEW ONES?

Question: Which questioning strategy/ies does/do the exchange of thoughts above illustrate?

A. Funneling

B. Sowing and reaping

C. Nose-dive

D. Extending and lifting

32. Which is NOT a sound purpose for asking questions?

A. To probe deeper after an answer is given.

B. To discipline a bully in class.

C. To remind students of a procedure.

D. To encourage self-reflection.

33. The main purpose of compulsory study of the Constitution is to __________

A. develop students into responsible, thinking citizens

B. acquaint students with the historical development of the Philippine Constitution

C. make constitutional experts of the students

D. prepare students for law-making

34. A child who gets punished for stealing candy may not steal again immediately. But this does not mean that the child may not steal again. Based on Thorndike's theory on punishment and learning, this shows that __________

A. punishment strengthens a response

B. punishment removes a response

C. punishment does not remove a response

D. punishment weakens a response

35. As a teacher, you are a reconstructionist. Which among these will be your guiding principle?

A. I must teach the child every knowledge, skill, and value that he needs for a better future.

B. I must teach the child to develop his mental powers to the full.

C. I must teach the child so he is assured of heaven.

D. I must teach the child that we can never have real knowledge of anything.

36. How can you exhibit expert power on the first day of school?

A. By making them feel you know what you are talking about.

B. By making them realize the importance of good grades.

C. By reminding them your students your authority over them again and again.

D. By giving your students a sense of belonging and acceptance.

37. A sixth grade twelve-year old boy comes from a dysfunctional family and has been abused and neglected. He has been to two orphanages and three different elementary schools. The student can decode on the second grade level, but he can comprehend orally material at the fourth or fifth grade level. The most probable cause/s of this student's reading problem is/are __________.

A. emotional factors

B. poor teaching

C. neurological factors

D. immaturity

38. Teacher Q does not want Teacher B to be promoted and so writes an anonymous letter against Teacher B accusing her of fabricated lies Teacher Q mails this anonymous letter to the Schools Division Superintendent. What should Teacher Q do if she has to act professionally?

A. Submit a signed justifiable criticism against Teacher B, if there is any.

B. Go straight to the Schools Division Superintendent and gives criticism verbally.

C. Hire a group to distribute poison letters against Teacher B for information dissemination.

D. Instigate student activists to read poison letters over the microphone.

39. In self-directed learning, to what extent should a teacher's scaffolding be?

A. To a degree the student needs it.

B. None, to force the student to learn by himself.

C. To the minimum, to speed up development of student's sense of independence.

D. To the maximum, in order to extend to the student all the help he needs.

40. If your Licensure Examination Test (LET) items sample adequately the competencies listed in the syllabi, it can be said that the LET possesses __________ validity.

A. concurrent

B. construct

C. content

D. predictive

41. Which guideline must be observed in the use of prompting to shape the correct performance of your students?

A. Use the least intrusive prompt first.

B. Use all prompts available.

C. Use the most intrusive prompt first.

D. Refrain from using prompts.

42. It is not wise to laugh at a two-year old child when he utters bad word because in his stage he is learning to __________.

A. consider other's views

B. distinguish sex differences

C. socialize

D. distinguish right from wrong

43. In a treatment for alcoholism, Ramil was made to drink an alcoholic beverage and then made to ingest a drug that produces nausea. Eventually, he was nauseated at the sight and smell of alcohol and stopped drinking alcohoLWhich theory explains this?

A. Operant conditioning

B. Social Learning Theory

C. Associative Learning

D. Attribution Theory

44. Which is a true foundation of the social order?

A. Obedient citizenry

B. The reciprocation of rights and duties

C. Strong political leadership

D. Equitable distribution of wealth

45. As a teacher, what do you do when you engage yourself in major task analysis?

A. Test if learning reached higher level thinking skills.

B. Breakdown a complex task into sub-skills.

C. Determine the level of thinking involved.

D. Revise lesson objectives.

46. In instructional planning it is necessary that the parts of the plan from the first to the last have __________.

A. clarity

B. symmetry

C. coherence

D. conciseness

47. In a study conducted, the pupils were asked which nationality they preferred, if given a choice. Majority of the pupils wanted to be Americans. In this case, in which obligation relative to the state, do schools seem to be failing? In their obligation to __________.

A. respect for all duly constituted authorities

B. promote national pride

C. promote obedience to the laws of the state

D. instill allegiance to the Constitution

48. The best way for a guidance counselor to begin to develop study skills and habits in underachieving student would be to __________.

A. have these underachieving students observe the study habits of excelling students

B. encourage students to talk about study habits from their own experiences

C. have them view film strips about various study approaches

D. give out a list of effective study approaches

49. What is most likely to happen to our economy when export continuously surpasses importis a thought question on __________.

A. creating

B. relating cause-and-effect

C. synthesizing

D. predicting

50. Direct instruction is for facts, rules, and actions as indirect instruction is for __________, __________, __________.

A. hypotheses, verified data and conclusions

B. concepts, patterns and abstractions

C. concepts, processes and generalizations

D. guesses, data and conclusions

51. Why should a teacher NOT use direct instruction all the time?

A. It requires much time.

B. It requires use of many supplementary materials.

C. It is generally effective only in the teaching of concepts and abstractions.

D. It reduces students engagement in learning.

52. The principle of individual differences requires teachers to __________.

A. give greater attention to gifted learners

B. provide for a variety of learning activities

C. treat all learners alike while in the classroom

D. prepare modules for slow learners in class

53. Which questioning practice promotes more class interaction?

A. Asking the question before calling on a student.

B. Focusing on divergent question

C. Focusing on convergent questions.

D. Asking rhetorical questions.

54. Teacher F wanted to teach the pupils the skill to do cross stitching. Her check up quiz was a written test on the steps of cross stitching. Which characteristic of a good test does it lack?

A. Scorability

B. Reliability

C. Objectivity

D. Validity

55. The attention to the development of a deep respect and affection for our rich cultural past is an influence of __________.

A. Confucius

B. Hegel

C. Teilhard de Chardin

D. Dewey

56. How can you exhibit legitimate power on the first day of school?

A. By making your students feel they are accepted for who they are.

B. By informing them you are allowed to act in loco parentis.

C. By making them realize the importance of good grades.

D. By making them feel you have mastery of subject matter.

57. Study this group of tests which was administered with the following results, then answer the question.

Subject Mean  SD  Ronnels's Score

Math  56  10  43

Physics  41  9  31

English  80  16  109

In which subject(s) did Ronnel perform most poorly in relation to the group's performance?

A. English

B. English and Math

C. Math

D. Physics

58. Which holds true to standardized tests?

A. They are used for comparative purposes

B. They are administered differently

C. They are scored according to different standards

D. They are used for assigning grades

59. Study this group of tests which was administered with the following results, then answer the question.

Subject Mean  SD  Ronnels's Score

Math  56  10  43

Physics  41  9  31

English  80  16  109

In which subject(s) were the scores most homogenous?

A. Math

B. English

C. Physics

D. Physics and Math

60. What can be said of Peter who obtained a score of 75 in a Grammar objective test?

A. He answered 75 items in the test correctly.

B. He answered 75% of the test items correctly.

C. His rating is 75.

D. He performed better than 5% of his classmates.

61. Quiz is to formative test while periodic is to __________

A. criterion-reference test

B. summative test

C. norm-reference test

D. diagnostic test

62. Which applies when skewness is zero?

A. Mean is greater than the median

B. Median is greater than mean

C. Scores have three modes

D. Scores are normally distributed

63. Value clarification as a strategy in Values Education classes is anchored on which philosophy?

A. Existentialism

B. Christian philosophy

C. Idealism

D. Hedonism

64. Out of 3 distracters in a multiple choice test item, namely B, C, and D, no pupil chose D as answer. This implies that D is __________

A. an ineffective distracter

B. a vague distracter

C. an effective distracter

D. a plausible distracter

65. All men are pretty much alike. It is only by custom that they are set apart, said one Oriental philosopher. Where can this thought be most inspiring?

A. In a multi-cultural group of learners

B. In multi-cultural and heterogeneous groups of learners and indigenous peoples' group

C. In a class composed of indigenous peoples

D. In heterogeneous class of learners

66. Which group of philosophers maintainthattruth exists in an objective order that is independent of the knower?

A. Idealists

B. Pragmatists

C. Existentialists

D. Realists

67. Each teacher is said to be a trustee of the cultural and educational heritage of the nation and is, under obligation to transmit to learners such heritage. Which practice makes him fulfill such obligation?

A. Use the latest instructional technology.

B. Observe continuing professional education.

C. Use interactive teaching strategies.

D. Study the life of Filipino heroes.

68. Teacher B engages her students with information for thorough understanding for meaning and for competent application. Which principle governs Teacher B's practice?

A. Contructivist

B. Gestalt

C. Behaviorist

D. Cognitivist

69. In which competency do my students find the greatest difficulty? In the item with a difficulty index of __________.

A. 0.1

B. 0.9

C. 0.5

D. 1.0

70. Rights and duties are correlative. This means that __________.

A. rights and duties regulate the relationship of men in society

B. rights and duties arise from natural law

C. each right carries with it one or several corresponding duties

D. rights and duties ultimately come from God

71. Which describes norm-referenced grading?

A. The performance of the group

B. What constitutes a perfect score

C. The students' past performance

D. An absolute standard

72. Teacher U teaches to his pupils that pleasure is not the highest good. Teacher's teaching is against what philosophy?

A. Realism

B. Hedonism

C. Epicureanism

D. Empiricism

73. Standard deviation is to variability as mode to __________.

A. level of difficulty

B. discrimination

C. correlation

D. central tendency

74. Which test has broad sampling of topics as strength?

A. Objective test

B. Short answer test

C. Essay test

D. Problem type

75. I combined several subject areas in order to focus on a single concept for inter-disciplinary teaching. Which strategy/method did I use?

A. Problem-entered learning

B. Thematic instruction

C. Reading-writing activity

D. Unit method

76. Which teaching activity is founded on Bandura's Social Learning Theory?

A. Lecturing

B. Modeling

C. Questioning

D. Inductive Reasoning

77. For which may you use the direct instruction method?

A. Become aware of the pollutants around us.

B. Appreciate Milton's Paradise Lost.

C. Use a microscope properly.

78. We encounter people whose prayer goes like this: "O God, if there is a God; save my soul, if I have a soul" From whom is this prayer?

A. Stoic

B. Empiricist

C. Agnostic

D. Skeptic

79. What measure of central tendency does the number 16 represent in the following data: 14, 15, 17, 16, 19, 20, 16, 14, 16?

A. Mode

B. Median

C. Mode and median

D. Mean

80. Availment of the Philippine Education Placement Test (PEPT) for adults and out-of-school youths is in support of the goverriment'S educational program towards __________.

A. equitable access

B. quality

C. quality and relevance

D. relevance

81. Which one can best evaluate students' attitudinal development?

A. Essay test

B. Portfolio

C. Observation

D. Short answer test

82. Which is a form of direct instruction?

A. Discovery process

B. Problem solving

C. Programmed instruction

D. Inductive reasoning

83. History books used in schools are replete with events portraying defeats and weaknesses of the Filipino as a people. How should you tackle them in the classroom?

A. Present them and express your feelings of shame.

B. Present facts and use them as means in inspiring your class to learn from them.

C. Present them and blame those people responsible or those who have contributed.

D. Present them as they are presented,and tell the class to accept reality.

84. Standard deviation is to variability as mean is to __________.

A. coefficient of correlation

B. central tendency

C. discrimination index

D. level of difficulty

85. A mother gives his boy his favorite snack everytime the boy cleans up his room. Afterwards, the boy cleaned his room everyday in anticipation of the snack. Which theory is illustrated?

A. Associative Learning

B. Classical Conditioning

C. Operant Conditioning

D. Pavlonian Conditioning

86. Bruner's theory on intellectual development moves from enactive to iconic and symbolic stages. In which stage(s) are diagrams helpful to accompany verbal information?

A. Enactive and iconic

B. Symbolic

C. Symbolic and enactive

D. Iconic

87. Teacher A discovered that his pupils are very good in dramatizing. Which tool must have helped him discover his pupils' strength?

A. Portfolio assessment

B. Performance test

C. Journal entry

D. Paper-and-pencil test

88. Which one can enhance the comparability of grades?

A. Using common conversion table for translating test scores in to ratings

B. Formulating tests that vary from one teacher to another

C. Allowing individual teachers to determine factors for rating

D. Individual teachers giving weights to factors considered for rating

89. In the Preamble of the Code of Ethics of Professional Teachers, which is NOT said of teachers?

A. LET passers

B. Duly licensed professionals

C. Possess dignity and reputation

D. With high-moral values as well as technical and professional competence

90. Which is the first step in planning an achievement test?

A. Define the instructional objective.

B. Decide on the length of the test.

C. Select the type of test items to use.

D. Build a table of specification.

91. A teacher who equates authority with power does NOT __________.

A. shame

B. develop self-respect in every pupil

C. retaliate

D. intimidate

92. Which is a major advantage of a curriculum-based assessment?

A. It is informal in nature.

B. It connects testing with teaching.

C. It tends to focus on anecdotal information on student progress.

D. It is based on a norm-referenced measurement model.

93. Which does NOT belong to the group of alternative learning systems?

A. Multi-grade grouping

B. Multi-age grouping

C. Graded education

D. Non-graded grouping

94. Theft of school equipment like tv, computer, etc. by teenagers in the community itself is becoming a common phenomenon. What does this incident signify?

A. Prevalence of poverty in the community.

B. Inability of school to hire security guards.

C. Deprivation of Filipino schools.

D. Community's lack of sense of co-ownership.

95. Which can effectively measure students' awareness of values?

A. Projective techniques

B. Moral dilemma

C. Likert scales

D. Anecdotal record

96. Based on Edgar Dale's Cone of Experience, which activity is closest to the real thing?

A. View images

B. Attend exhibit

C. Watch a demo

D. Hear

97. Teacher B is a teacher of English as a Second Language. She uses vocabulary cards, fill-in-the-blank sentences, dictation and writing exercises in teaching a lesson about grocery shopping. Based on this information, which of the following is a valid conclusion?

A. The teacher is reinforcing learning by giving the same information in, a variety of methods.

B. The teacher is applying Bloom's hierachy of cognitive learning.

C. The teacher wants to do less talk.

D. The teacher is emphasizing listening and speaking skills.

98. Helping in the development of graduates who aremaka-Diyosis an influence of

A. naturalistic morality

B. classical Christian morality

C. situational morality

D. dialectical morality

99. From whom do we owe the theory of deductive interference as illustrated in syllogisms?

A. Plato

B. Scorates

C. Aristotle

D. Pythagoras

100. Studies in the areas of neurosciences disclosed that the human brain has limitless capacity. What does this imply?

A. Some pupils are admittedly not capable of learning.

B. Every pupil has his own native ability and his learning is limited to this nativeabilty.

C. Every child is a potential genius.

D. Pupils can possibly reach a point where they have learned everything.

C

B

D

D

D

C

D

B

C

B

B

A

C

A

C

C

D

D

A

D

B

B

B

C

D

A

B

C

B

B

D

B

A

C

A

A

C

A

C

C

A

A

C

B

B

B

B

B

B

C

C

B

B

A

A

D

A

C

D

D

B

D

C

A

B

D

D

D

A

C

A

B

C

C

D

D

C

C

C

A

B

C

B

B

D

B

B

A

A

D

B

D

C

D

D

B

A

B

A

C

------------


PROFESSIONAL EDUCATION BULLETS 

- - - - - - - - - - - - - - - - - - - - - - - - - - - - - - 

1.The most reliable measure of central tendency when there are extreme scores

●Median

.

2. Iah's score from her LET are the following: 92, 88, 91. What is the median?

●91

3. What can be inferred from a low standard deviation? ●Scores are homogenous

4. A high negative discrimination index means that: ●More from the bottom group answered the test question correctly

5. When a student scored P80 on a single test, that means: ●He scored higher than 80 percent of the class

6. Which of the following is an accurate depiction of an authentic assessment? ●Performing first-aid measures to scald burns

7. Which of the following is a characteristics of a norm-referenced testing? ●The performance depends on the scores of his batch mates

8. Which of the following is a correct statement about validity and reliability? ●A valid test is always reliable

9. Which of the following is a characteristics of a norm-referenced testing? ●The performance depends on the scores of his batch mates

10. A difficulty index of .92 means that the item is: ●Very easy

11. Which of the following is an accurate depiction of an authentic assessment? ●Performing first-aid measures to scald burns

12.Which of the following is an example of a formative test? ●Quiz

13. Which of the following types of tests is most vulnerable to biases? ●Essays

14. A leptokurtic distribution signifies that: ●All scores are average

15. If a distribution is skewed to the right, this implies that: ●The scores are very low

16. Which of the following is the best method to imcrease the reliablity of a test material? ●Increase the number of items

17. This thinking strategy refers to narrowing down ideas from big concept to smaller ones? ●Convergent Thinking

18. This refers to the collection of works, artifacts, and pieces of a student and may serve as a basis for assessment: ●Portfolio

19. Which of the following does Not describe a holistic rubric in scoring ●It uses at least 3 or more basis

20. Which of the following types of tests is most vulnerable to biases? ●Essays

21. Withitness means: ●That the teacher knows everything that happens around the four corner of the classroom

22. When a teacher jumps from one topic to another without assessing of the students are ready to absorb the instructions, this practice depicts: ●Thrust

23. What can be inferred from a low standard deviation? ●Scores are homogenous

24. If a distribution is skewed to the right, this implies that: ●The scores are very low

25. When a teacher is able to perform multiple, different activities at the same, this shows: ●Overlapping

26. Which is an incorrect practice in test construction? ●When using numbers as options, arrange them in a descending order

27. Under the new version of Bloom's Taxonomy of conitive processes, which of the following belongs to the top level? ●Generating

28. Which is the basic form of cognitive process according to Bloom's? ●Knowledge

29. Which of the following forms of punishment is least likely to affect students negatively? ●Surprise quiz

30. The following are examples of extrinsic motivation, except? ●Community service

31. This is also known as the Enhanced Basic Education Act of 2013 ●K-12

32. Subject matter, Activities, Evaluation, Assignment, Sequence of a lesson plan ●Determine the objectives

33. A student makes the teachers and his classmates busy and asks everyone to give him special attention. What is the student's hidden message? ●The students wants to feel connected

34. Which of the following serves as a pre-requisite to employment and will ensure that only competent teachers will be granted privilege to teach to schools? ●Licensure and registration

35. Which of the following correctly describes inductive reasoning? ●Arriving to a main idea demo smaller topics

36. Which of the following laws prescribed licensure examination for teachers and will strengthen the regulation of the practice of teaching in the Philippines? ●RA 7836

37. Which of the following will disqualify a teacher to become a member of the board of professional teachers? ●A naturalized Filipino citizen that has 15 years of continuous teaching experience

38. Which of the following shall receive the highest budgetary allocation according to the Philippine Constitution? ●Education

39. According to the law, she can be referred to as a para-teacher. Which of the following is not correct about para-teachers? ●These teacher failed to reach the minimum GWA of 75 but have grades below 75

Kylie scored 74.80 from the recent LET.

40. Which of the following is most likely be related to existentialism? ●Free choice

41. Which of the following is NOT true about periodic merot examination for teachers according to RA 7836? ●This shall encourage continuing personal growth and development

42. While teaching Mathematics, Teacher Janus noticed that his students show interests on dancing. He then decides to change the topic and teaches concept about dancing and shows to them dance perdormances from the internet. What philosophy of education is shown? ●Progressivism

43. If a teacher wishes to enjoy study leave, she will get how many percent of her salary while on study leave? ●60%

44. What is the focus of education during the Commonwealth Period? ●Nationalism

45. Which of the following is the focus of Spartan Education? ●Military training

46. Teacher Vanessa receives a death threat because she gave her student a failing grade. She knows that passing the student will make her less credible as a teacher but will save herself from being harmed. She then decided to pass the student to avoid danger. Which of the following is observed by Teacher Vanessa? ●Principle of Double Effect

47. Which of the following actions of a teacher violated the Code of Ethics for Professional Teachers?●Having practice prejudice and eliminating discrimination against any learner

48. This type of conscience makes the person see that he sins but actually he/she does not. ●Scrupulous

49. K-12 curriculum is what type of subject-centered curriculum? ●Spiral

50. Which is the closest to the real thing? ●Performing real life task

51. Which of the following helps the learner to retain information best? ●Doing

52. A globe is an example of a ●Model

53. Which of the four pillars of learning focuses on the competencies of a certain individual on a particular skill? ●Learning to do

54. Which of the following involves enactive representations according to Bruner? ●Action-based

55. Which does not show acculturation? ●Learning from your American parents their traditions

56. A Venn Diagram is most likely to be used when: ●Discussing the similarities and differences of prokaryotic and eukaryotic cells

57. Teacher Iah wants to show her students the percentage of Math major, science major, social science major, english major and tle major based on the whole population of BSED students in the school where she teaches. Which of the following types of graph should she use? ●Circle graph

58. Which of the following traits of Filipino can be considered both a negative and a positive trait?●Pakikisama mentality

59. he ability of a child to realize that 1 liter of bottled water has the same quantity even if the water will be placed on a plastic pitcher or a pale is called: ●Conservation

60. Ana is a 13 year old high school student who thinks that death penalty should be implemented in her own countries as she believes that there are other ways to punish oppressors and teach them to be better citizens. She should be under what stage of Piaget's theory? ●Formal operational

61. Lawrence Kohlberg focuses more on which of the following aspects of child development?●Morality

62. Which does not show acculturation? ●Learning from your American parents their traditions

63. A toddler wishes to go the restroom alone and wishes to care for himself if not treated properly will have develop or lead to which of the following psychosocial stages based on Erikson's model? ●Doubt

64. This is known as a girl's psychosexual competition with her mother for the possession of her father. ●Electra Complex

65. The parts of human personality in which instinct and primary processes manifest: ●Id

66. Which of the following shall receive the highest priority accordin to Abraham Maslow's model on ●Warmth humanistic needs?

67. In Grace Goodell's Reading Skills Ladder, which of the following takes precedence? ●Basic sight words

68. Which of the following is not true about Language Acquisition among children? ●Chomsky explains in his model thay children acquired language solely through exposure.

69. This model shows reading as an active process that depends on reader characteristics, the text, and ●Interactive the reading situation

70. A child who has spatial intelligence will most likely enjoy which of the followong activities? ●Solving puzzles

71. It is the average, most commonly used and in greatly affected by extreme scores ●Mean

72. Who is the proponent of respondent conditioning? ●Ivan Pavlov

73. It is the middlemost of measures of central tendency and most reliable when there is extreme scores ●Median

74. Which of the following teacher most likely demonstrates Bandura's work in teaching? ●She shows the student how a dance should be performed.

75. It is the most frequently used measures of central tendency ●Mode

76. Which of the following refers to the law of effect by Thorndike? ●If the end result will be beneficial to the student he/she will probably perdorm well

77. It is the highest score minus the lowest score and is the simplest of measures of variability ●Range

78. ZPD is concept popularized by: ●Vygotsky

79. It is how spread the scores are from the mean and most reliable measures of variability ●Standard Deviation

79. It is the square of standard deviation ●Variance

80. Very easy/reject ●0.81-1.00

81. Very difficult/reject ●0-0.20

82. Easiness ●Difficulty index

83. Differentiate Upper group to lower group ●Discrimination index

84. Difficult/revise ●0.21-0.40

85. More from upper group/retain ●Positive Discrimination index

86.Moderate/retain ●0.41-0.60

87. Easy/revise ●0.61-0.80

88. More from lower group/reject ●Negative discrimination index

89. Cannot determine/reject ●Zero discrimination index

90. Measures what it intends to measure ●Validity

91. Consistency ●Reliability

92. Standard, criteria, specific target ●Criterion referenced

91. Others, class, batch mates ●Norm referenced

92. Scattered, far from the mean, heterogenous ●High standard deviation

93. Pen and paper, multiple choice, cognitive ●Traditional assessment

94. Divide by 9. Median is S5 ●Stanines

95. Clustered, near from the mean, homogenous ●Low standard deviation

96. Real-life application, holistic ●Authentic Assessment

97. Peakedness ●Kurtosis

98. Before instruction, strengths and weaknesses ●Diagnostic

99. Divide by 100. Median is P50 ●Percentile

100. Normal curve, bell-shaped, most scores-average, few scores are high and low ●Mesokurtic

101. Taller, more peaked, almost if not all scores are average ●Leptokurtic

102. Divide by 10. Median is D5 ●Decile

103. During instruction progress, gaps, quizzes ●Formative

104. Flat curve, scores are heterogenous ●Platykurtic

105. Divide by 4. Median is Q2 ●Quartile

106. Afetr instruction, evaluative learning ●Summative

107. Not biased, multiple choice, matching type, wide level of objectives, guessing ●Objective

108. Guide for scoring ●Rubrics

109. Biased, essay, wide sampling of ideas, bluffing ●Subjective

110. Single basis ●Holistic

111. Memorization ●Knowledge

112. Multiple basis ●Analytical

113. Understanding ●Comprehension

114. Use ●Application

115. Classroom Management Model ●Kounin

116. Breaking down ●Analysis

117. Eyes at the back ●Withitness

118. Synthesis

●Putting together

119. Multi-tasking ●Overlapping

120. Jumping from previous topic to new topic and vice versa ●Flip-flop

121. Judgment ●Evaluation

122. Narrowing ●Convergent thinking

123. Not able to return ●Truncation

124. The students are not ready ●Thrust

125.Widening ●Divergent thinking

126. Increase a response ●Reinforcement

127.Weakens a response ●Punsihment

128. General to specific ●Deductive reasoning

129. Outside ●Acculturation

130. Specific to general ●Inductive reasoning

131. Inside ●Enculturation

132. External, shallow, money ●Extrinsic motivation

133. Cognitive development ●Piaget

134. Internal, noble, social work ●Intrinsic motivation

135. Senses and motor (0-2) ●Sensorimotor

136. Hidden message: notice me

Requires special treatment

Keeps others busy ●Attention seeking

137. Egocentric, children tend to see only their point of view(2-7) ●Pre-operational

138. Hidden message: i am hurting

Get even

Violent ●Revenge seeking

139. (7-12) ●Concrete operational

140. 12-above

Abstract reasoning ●Formal operational

141. Hidden message: i want to help

Gets bossy ●Power seeking

142. Classical conditioning or respondent conditioning ●Pavlov

143. Hidden message: show me how

Refuses to participate ●Isolation/Withdrawal

144. Operant conditioning ●Skinner

145.Back to basics ●Essentialism

146. Social learning theory

Modelling

Imitation ●Albert Bandura

147.Traditional ●Perennialism

148. Trust vs. Mistrust ●Infant

149. Free choice ●Existentialism

150. Autonomy vs. Shame and doubt ●Toddler

151. Change ●Progressivism

152. Initiative vs. Guilt ●Preschool

153. Social change ●Social Reconstructionism

154. Industry vs. Inferiority ●School age

155. Practical use ●Pragmatism

156. Identity vs. Role confusion ●Adolescence

157. Labor/vocational training ●Japanese

158. Values, beliefs, affective, attitude ●Idealism

159. 2 options but each has both positive and negative effect ●Double effect

160. Senses ●Empiricism

161. 2 negative options but the other one is less evil ●Lesser evil

162. Intimacy vs. Isolation ●Young adult

163. With knowledge and intention ●Formal cooperation

164. Without knowledge and intention ●Material cooperation

165. Create new knowledge ●Constructivism

166.Pleasure ●Hedonism

167. Generativity vs. Stagnation ●Middle adult

168. More people will benefit ●Utilitarianism

169. Environment ●Behaviorism

170. Sure ●Certain

171. Ego integrity vs. Despair ●Late adult

172. Unsure ●Doubtful

173. Survival ●Pre-spanish

174. Hypocrite ●Pharisaical

175.Mouth ●Oral

176. Religion ●Spanish

177. Insensitive ●Callous

178. Thinks shes right but shes not ●Lax

179.Anus ●Anal

180. Thinks shes wrong but shes not ●Scrupulous

181. Nationalism ●Commonwealth

182.Sex organs ●Phallic

183. 10% ●Read

184. 20% ●Hear

185. Free basic education ●American

186. None ●Latentcy

187. 30% ●See

188. 50% ●Hear and see

189. Sex organs through opposite sex ●Genital

190. Reader to book ●Top down

191. 70% ●Say and write

192. Book to reader ●Bottom up

193. 90% ●Do

194. Action ●Enactive

195. Active process ●Interactive

196. Images ●Iconic

197. Language ●Symbolic

198. Trend or progress ●Line graph

199. Comparison ●Bar graph

200.Percentage ●Circle or pie graph

201. A process by which a conditioned response is lost. ●Extinction

NCBTS -National Competency based -teacher standards

🔥Prof Ed: Additional Notes 

⭐Which type of visual shows the actual object under study? – 

👉Realistic

⭐Which perspective views the learner as actively creating meaning? –

👉 Constructivism

⭐Why is it important for teachers to understand learning theory? -

👉 Theory informs practice

⭐The evaluation activity in a lesson plan should answer the following question: 

👉How will you determine if students have achieved the learning objectives?

⭐What is the main principle of the discovery method? - 

👉Students learn best by doing

⭐The purpose of a motivation activity in a lesson plan is to: 

👉gain and maintain students' attention

⭐Which perspective says that learning should take place in an environment that resembles the real world with all its complexities? – 

👉Constructivism

⭐ISTE standards are 

👉technology standards for students and teachers

⭐Which of the following is an acceptable verb to use in objectives? – 

👉describe

⭐This theory believes that learning is largely determined by the external environment – 

👉behaviorism

⭐According to Marianne Torbet, which of the following is not a game inclusion factor? – 

👉Competition

⭐Which of the following are the procedures and actions used to help students meet stated objectives? -

👉 Methods 

⭐What type of lesson evaluation takes place during the planning of the lesson? –

👉 Formative

⭐Visuals that convey a concept by comparing one idea to another and implying a similarity are called 

👉analogic visuals

⭐Which is the physiological process in which sound waves enter the ear and are converted into electrical impulses that travel to the brain? –

👉 Hearing

⭐Which of the following is NOT a fair use guideline? -

👉 The cost of the item in question

⭐The information activity in a lesson plan should answer the following question: 

👉How will you help students see relationships among ideas?

⭐According to the authors of Children's Ministry that Works, in choosing games for children, which of the following is not an important factor? - 

👉The games should be competitive

⭐Which of the following are evidence of "the digital divide": i.e., the gap in equitable use of computer technology among student groups? - 

👉Computers used for drill and skill only in low income districts

⭐Which of the following experience is most closely associated with the constructivist perspective of learning? -

👉 Student teaching

⭐Sam brought his rock collection to class for his students to classify. Which type of media is being used? –

👉 Exhibit

⭐Which of the following is a three dimensional representation of a real object? –

👉 Model

⭐Which of the following is true regarding the shift in education in recent years? -

 ðŸ‘‰There has been a shift toward learner-centered instruction.

⭐Copyright law protects -

👉 the legal rights to original works

⭐The ability to accurately interpret and create visual messages is known as

👉 visual literacy

⭐Which of the following is a problem exacerbated by the prevalence of social media sites such as Facebook?

👉 - Cyber-bullying

⭐The purpose of an application activity or conclusion section in a lesson plan is to

👉 provide opportunity for practice and feedback.

⭐Which of the following is true regarding field trips?

👉 - They are a form of enactive learning.

⭐The PIE model of classroom instruction stands for

👉: plan, implement, evaluate

⭐Which of the following objectives contains acceptable criteria? - 

👉Students will compose a paragraph with no more than two errors.

⭐This theory uses the computer as a model for the way humans think -

👉 informational processing

⭐Persistence of vision is a phenomenon whereby - 

👉the brain continues to see the image for a fraction of a second after the image is cut off

⭐Interpreting a visual is also known as

👉 decoding

⭐What is the difference between informational processing theory and constructivism? - 

👉The first says that knowledge is objective and represents experience; the second that knowledge is subjective and depends on the learner's interpretation of experience.

⭐An enduring change in human behavior or performance resulting from practice or experience is the

👉 definition of learning

⭐Which of the following is NOT a step in the scientific process, often used in the problem solving method? -

👉 explore the question

⭐The rule of thirds tells us that elements should be arranged 

👉along imaginary lines dividing your visual in three

⭐Sam borrowed ½ of an engine with the inners workings exposed from a local tech school. Which type of media is this? –

👉 Cutaway

⭐Which of the following is NOT a criteria for selecting and/or creating instructional materials? -

👉 Are all materials original?

⭐Applying technological processes and tools to solve problems of teaching and learning is the definition of

👉 educational technology

⭐Charts and graphs are what type of visuals? –

👉 Organizational

⭐In general, which gender seems to be more concerned with social relationships?

👉 – Girls

⭐The conclusion section of the lesson plan serves to provide opportunity for

👉 practice and feedback

⭐Which of the following is not one of the three categories of criteria in written objectives? – 

👉Conditions

⭐Which learning theory uses the ABC model to explain the learning process? –

👉 Behaviorism

⭐According to the Pew Research Center, what category of internet users have increased the most from 2000 to 2011? -

👉 High school grads

⭐An organized set of principles meant to explain events is the definition of the word

👉 theory

⭐Creating a visual is also known as

👉 encoding

⭐The systematic application of scientific or other organized knowledge to practical tasks is the definition of

👉 technology

⭐Which of the following is the channel of communication within the lesson? – 

👉Media

⭐Adaptive Learning - uses computers as interactive instructional devices; adapts the difficulty or material to the needs of the

👉 students

⭐Asynchronous Learning - students can learn different things at different times and at different places due to the internet

⭐Blended Learning - a teaching method that combines traditional classroom instruction with online or mobile learning activities

⭐Cloud - metaphor for on-demand storage space or computing power managed by a third party

⭐Digital Citizenship - making good use of the Internet and having knowledge of how to operate web-connected devices safely while online; interacting respectfully with others

⭐Digital Divide - the large gap in technology use between two groups due to economic, racial, age, or gender lines

⭐Flipped Classroom - traditional face-to-face lecture is recorded and watched and posted online for students to watch online; teachers use class time to solve problems and interact with students

⭐Gamification - the making of boring, every day, or ordinary activities into game-like activities

⭐M-Learning - Mobile Learning; any learning that takes place on a mobile device

⭐MOOC - Massively Open Online Course

⭐Podcast - similar to a radio show but distributed via the internet rather than radio waves

⭐QR Code - like a barcode with a link, when scanned by mobile device it takes you to that link

⭐Learning Management System (LMS) - Digital resource for class discussion, document management, homework submission, and course scheduling.

⭐Differentiated Learning - Presenting learning materials in ways that match students' varying learning styles or levels.

⭐Digital Storytelling - Using web-based tools to create and tell stories; with some mixture of digital images, text, recorded audio narration, video clips and/or music

⭐E-Books - Books that are completely digital and are usually read on computers or e-readers.

⭐Flipped Classroom - Students access the directed teaching at home, through videos, and spend in-class time applying skills through guided practice and activities.

⭐Gamification - Using game design and mechanics to drive motivation and increase engagement in learning.

⭐Individualized Learning - When a group of students all receive the same content but work through it at their own pace.

⭐Lifelong Learning - Lifelong learning continues education informally for personal enrichment, usually after finishing formal education.

⭐Open Educational Resources (OER) - Digital materials available for reuse and repurposing in teaching, researching, and learning.

⭐Personalized Learning - its learning entirely geared toward the individual student.

⭐Digital Cloud - On-demand storage space, applications or services offered over the internet, managed off-site.

⭐Digital Citizenship - Navigating the digital world safely, responsibly, and ethically.

⭐Professional Learning Community (PLC) - Community focused around a particular topic, interest, or subject in order to share best practices and resources.

⭐Trouble Ticket - An online form used to report problems.

Post a Comment

0Comments

https:www.majait.net

Post a Comment (0)